You are on page 1of 45

wrwftAuditing Problems

watitiw
1. Draftwqbejk income statement of Raffles Inc. is as follows:
Sales
Cost of sales
Opening inventory
Purchases
Closing inventory
Gross profit
Expenses
Profit
wwefjbewtrwt

P700,000
60,000
430,000
90,000

400,000
300,000
200,000
P100,000

How much is the correct profit after the following adjustments?


(1) Closing inventory includes goods costing P20,000 which are expecthrtjhtyjtyed to realize at P19,000.
(2) A customer has taken legal action for damages of hgjP50,000 against Raffles. The lawyer of Raffles
has advised the customer that he has a 25% chance of succeskj.
(3) After the balance sheet date, a vehicle was damaged in an accident. The carrying amount of the vehicle
was P6,000. It was not insured.
(4) Raffles has sued one of its competitors for P60,000. The chance of Raffles winning the case is 75%.
The outcome will be known in three months.
A
.
B.
C.
D
.

P159,000
P103,000
P99,000
P49,000

Solution: C is correct
Unadjusted profit
(1) Write down of inventory to its NRV (20,000 19,000)
(2) Loss not probable no adjustment
(3) Non-adjusting event after the balance sheet date - no
adjustment
(4) Gain not virtuallycertain no adjustment
Adjusted profit

P100,000
(1,000)
P99,000

Page 1 of 45

2. Zod Company reports the following information as of December 31:


Sales revenue
Cost of goods sold
Operating expenses
Unrealized gain on available-for-sale securities

P800,000
600,000
90,000
30,000

How much should the company report as total comprehensive income?


A
.
B.
C.
D
.

P80,000
P110,000
P140,000
P200,000

Solution: C is correct
Sales revenue
Less: Cost of goods sold
Less: Operating expenses
Amount taken to P&L
Amount taken to OCI
Total comprehensive income

P800,000
600,000
90,000
P110,000
30,000
P140,000

3. You are assigned to the audit of Canon Company who spent P12,000,000 during the current year
developing its new software package. Of this amount, P4,000,000 was spent before it was at the
application development stage and the package was only to be used internally. The package was
completed during the year and is expected to have a four year useful life. Canon Company has a policy of
taking a full-year's amortization in the first year. After the development stage, P50,000 was spent on
training employees to use the program. What amount should Canon Company report as an expense for the
current year?
A
.
B.
C.
D
.

P2,012,500
P2,050,000
P6,012,500
P6,050,000

Solution: D is correct
Amount spent before the development stage
Amount spent on training employees
Amortization of the software package
(12M 4M) / 4 years
Total expense to be recognized

P4,000,000
50,000
2,000,000
P6,050,000

4. You were assigned to audit the factory accounts of Ministops Corporation for the year ended December
31, 2012. The following data were gathered: Manufacturing cost totalled P900,000. Cost of goods
manufactured was P800,000 of which factory overhead was 75% of direct labor. Overhead was 25% of
total manufacturing cost. Beginning work-in process inventory was 60% of ending work-in-process
Page 2 of 45

inventory. Manufacturing costs for the year ended December 31, 2012 submitted to you by the factory
accountant were as follows:
Raw materials used
Direct labor
Factory overhead
Total

P400,000
275,000
225,000
P900,000

Assume cost percentage relationships as stated were correct. Which of the following adjusting journal
entries is correct?
A. Raw materials used
Direct labor
B. Direct labor
Raw materials used
C. Raw materials used
Direct labor
D. Direct labor
Raw materials used

P25,000
P25,000
P25,000
P25,000
P50,000
P50,000
P50,000
P50,000

Solution: B is correct
Factory overhead (75% of direct labor)
Divided by 75%
Direct labor, per recomputation
Direct labor, unadjusted
Adjustment to direct labor

P225,000
75%
300,000
275,000
P25,000

5. On June 1, 2008, Psy Corporation purchased as a long term investment 6,000 of the P1,000 face value, 8%
bonds of Lyre Corporation. Psy Corporation has the positive intention and ability to hold these bonds to
maturity. The bonds were purchased to yield 10% interest. Interest is payable semi-annually on
December 1 and June 1. The bonds mature on June 1, 2014. On November 1, 2009, Psy Corporation sold
the bonds for a total consideration of P5,887,500. Based on the above and the result of your audit,
determine the following: (Round off present value factors to four decimal places) - the purchase price of
the bonds on June 1, 2008 is
A
.
B.
C.
D
.

P5,467,992
P5,467,992
P5,545,104
P5,436,894

Solution: A is correct
PV of the principal
(0.5568 x 6,000,000)
PV of the interest
(8.8633 x 6,000,000 x 4%)

P3,340,800

2,127,192
P5,467,992
6. The Tiger Corporation included the following in its unadjusted trial balance as of December 31, 2012:
Inventory, 12/31/11
Purchases

P19,450,000
127,850,000
Page 3 of 45

Additional information:

The inventory at December 31, 2012 was counted at a cost of P8.5 million. This includes P500,000 of
slow moving inventory that is expected to be sold for P300,000.
Sales include P8 million for goods sold in December 2012 for cash to Beer Finance Company. The cost
of these goods was P6 million. Beer Finance Company has the option to require Tiger to repurchase
these goods within one month of year-end at their original selling price plus a facilitating fee of
P250,000.
The Companys accounting policy is to present cost of writing down inventory to NRV as part of cost
of sales.

The cost of sales for the year ended December 31, 2012 is
A P138,800,000
.
B. P133,000,000
C. P132,800,000
D P139.000.000
.
Solution: B is correct
Amount in thousands:
Inventory, beginning
Add: Purchases
Less: Inventory, ending
Cost of sales, unadjusted
Add: to adjust to net realizable value the items in ending inventory
Less: Item sold under repurchase agreement
Cost of sales, adjusted

P19,450
127,850
8,500
138,800
200
6,000
P133,000

7. Inventory per count on December 31, 2012 as reported by WS Company was at P560,000. The amount
does not include the following items:
A. Merchandise costing P27,500 received on January 3, 2013, and purchase invoice recorded on
January 5, 2013. Shipment was made on December 25, 2012,terms FOB destination.
B. Merchandise costing P20,000 was physically segregated in the warehouse and was set aside for
shipmentto a customer (shipment scheduled on January 2, 2013). This will be invoiced to the
customer by then at P28,000.
C. Merchandise received on January 7, 2013, costing P17,000 was entered in the purchases journal on
January 8, 2013. Invoice shipment was made FOB shipping point on December 30, 2012.
What is the adjusted balance of inventories to be presented in the 2012 statement of financial position?
A P597,000
.
B. P580,000
C. P578,000
D P587,500
.
Solution: A is correct
Page 4 of 45

Unadjusted inventory balance


Item (a) Goods received January 2013, under FOB Destination, thus item is
correctly excluded
Item (b) Goods still in warehouse but were not included in the count
Item (c) Goods received January 2013, under FOB shipping point. Items shipped
December 30, 2012
Adjusted inventory balance

P560,000
20,000
17,000
P597,000

8. Bohol Company leased office premises to Jill Company for a 5-year term starting January 2, 2013. Under the
terms of the lease, rent for the first year is P200,000 and rent for years 2 through 5 is P300,000 annually. As
an inducement to enter the lease, Bohol Company waives the first six month payments. Jill Company likewise
paid a P70,000 security deposit of which 80% is refundable at the end of the lease term. Furthermore,
contingent rent equal to 2% of sales in excess of P12,000,000 shall be paid by Jill Company. Bohol Company
incurred initial direct cost of P40,000 while JL Company paid P30,000 in costs in relation to the lease. In
2013, Jill Company reported sales of P13,000,000.
Rental expense to be included in Jill Companys 2013 income statement is
A 288,800
.
B. 268,800
C. 302,200
D 315,000
.
Solution: A is correct
Total rental payments for five years(P200,000/2) + (P300,000 x 4)
Divided by total years
Annual rental expense (fixed)
Contingent rent [(P13M P12M) x 2%]
Direct cost incurred(30,000 / 5)
Non-refundable portion of security deposit(P70,000 x 20%) / 5
Total rental expense

P1,300,000
5 years
P260,000
20,000
6,000
2,800
P288,800

The following information applies to the next two questions:


Listed below are four interbank cash transfers, indicated by the numbers 1, 2, 3, and 4, of a client
for late December 2010 and early January 2011:
Bank Account One
Bank Account Two
Disbursing Date

Receiving Date

(Month/Day)

(Month/Day)

Per Bank

Per Books

Per Bank Per Books

1.

12/31

12/30

12/31

12/30

2.

1/2

12/30

12/31

12/31

3.

1/3

12/31

1/21/2

4.

1/3

12/31

1/212/31

Page 5 of 45

9.

Based on the schedule of interbank transfers above, which of the cash transfers indicates an error in
cash cutoff at December 31, 2010?
a.

b.

c.

d.

Answer: C

10.

Based on the schedule of interbank transfers above, which of the cash transfers would appear as a
deposit in transit on the December 31, 2010 bank reconciliation?
a.

b.

c.

d.

Answer: D
Auditing Theory
1. Which of the following are the three principal methods in sample selection are specifically identified in PSA
530?
A. Statistical sampling, attribute sampling and haphazard sampling
B. Random number selection, systematic selection and haphazard selection
C. Sequential sampling, discovery sampling and statistical sampling
D. Sequential sampling, discovery sampling and random number selection
Answer: B
(PSA 530, Audit Sampling)
2. Which of the following is an appropriate consideration in auditors selection of sample size?
A. The auditor may select a voided or cancelled document in a sample. If the document has been properly
voided, treat the item as deviation.
B. If the auditor encounters missing documents and he is unable to determine whether control has been
properly performed, replace the document with another sample item.
C. The auditor may select a voided or cancelled document in a sample. If the document has been properly
voided, replace the document with another sample item
D. If the auditor encounters missing documents and he is unable to determine whether control has been
properly performed, treat the item as deviation and replace the document with another sample item.
Answer: C
(PSA 530, Audit Sampling)
Page 6 of 45

The auditor may select a voided or cancelled document in a sample. If the document has been properly voided,
the document must be replaced with another sample item. If the auditor encounters missing documents and he is
unable to determine whether control has been properly performed, the auditor must treat the item as deviation.
3. Fraudulent financial reporting involves intentional misstatements or omissions of amount or disclosures in the
financial statements to deceive financial statement users. Which of the following acts appropriately involves
fraudulent financial statements?
I.
II.
III.
IV.
V.
A.
B.
C.
D.

Manipulations, falsification or alteration of records or documents


Lapping of accounts receivable
Recording of transactions without substance
Stealing entitys assets such as cash, marketable securities and inventory
Intentional application of accounting policies
I and II
I, II , III and IV
I, III and V
All of the above

Answer: C
(PSA 240, Auditors Responsibility to Consider Fraud in an Audit of Financial Statements)
Fraudulent financial reporting involves intentional misstatements or omissions of amount or disclosures in the
financial statements to deceive financial statement users. This may involve:
I.
Manipulations, falsification or alteration of records or documents.
II.
Misrepresentation in or intentional omission of the effects of transactions from records or documents.
III.
Recording of transactions without substance and
IV.
Intentional application of accounting policies
4. Which of the following is not true with reference to the Code of Professional Ethics for CPAs?
A. Acting as advocate of an audit client in resolution of a dispute or litigation that involves amounts material
to the financial statements of the audit client impairs CPAs independence.
B. A firm or network firm should not provide accounting and bookkeeping services for an audit client that is
not a listed entity.
C. Fees are not to be regarded as contingent if these are fixed by a court
D. A professional accountant in public practice may enter into an arrangement for the purchase of the whole
or part of an accounting practice requiring payments to individuals formerly engaged in the practice.
Answer: B
(Code of Professional Ethics for CPAs)
According to Code of Professional Ethics for CPAs, a firm or network firm should not provide accounting and
bookkeeping services for an audit client that is a listed entity. The provision of such services may impair CPAs
independence, or at least give the appearance of impairing independence.
5. Which of the following acts is prohibited by the Code of Professional Ethics for CPAs?
A. The use of a firm name which includes the name of a retired partner
B. An announcement in a newspaper of the opening of a public accounting office
C. Engaging in civic activities during business hours
D. Accepting an engagement or employment which one cannot reasonable expect to compete or discharge
with professional competence

Page 7 of 45

Answer: D
(Code of Professional Ethics for CPAs)
6. When an independent auditors examination of financial statements discloses special circumstances that
make the auditor suspects that fraud may exist, the auditors initial course of action should be to
A. Recommend that the client pursue the suspected fraud to a conclusion that is agreeable to the auditor
B. Extend normal audit procedures in an attempt to detect the full extent of the suspected fraud
C. Reach an understanding with the proper client representative as to whether the auditor or the client is to
make the investigation necessary to determine if a fraud has in fact occurred
D. Determine whether the fraud, if in fact it does exist, might be of such a magnitude as to affect the auditors
report on the financial statements
Answer: D
(PSA 250, Consideration of Laws and Regulations in an Audit of Financial Statements, Section 58)
7. When an independent auditor decides that the work performed by internal auditors may have a bearing on the
nature, timing and extent of planned audit procedures, the independent auditor should evaluate the objectivity
of the internal auditors. Relative to objectivity, the independent auditor should
A.
B.
C.
D.

Consider the organizational level to which internal auditors report


Review the quality control program in effect for the internal audit staff
Examine the quality of the internal audit reports
Consider the qualifications of the internal audit staff

Answer: A
(PSA 610, Using the Work of Internal Auditors)
8. During an assessment of the risk associated with sales contracts and related commissions, which of the
following factors would most likely result in an expansion of the engagement scope?
A. An increase in product sales, along with an increase in commissions
B. An increase in sales returns, along with an increase in commissions
C. A decrease in sales commissions, along with a decrease in product sales
D. A decrease in sales returns, along with an increase in product sales
Answer: B
(Gleims Certified Internal Auditor Reviewer)
Trends described in A, C and D are compatible while trend in B are incompatible and may indicate inflated sales
figures.
9. Which of the following explanations suggests the least amount of relative risk stemming from a failure to
compare a purchase order to an approved price list?
A. A temporary employee processed the purchase order.
B. The comparison is not required by company policy.
C. The vendor is one used often by the company.
D. The director of the purchasing department approved the purchase order.
Answer: A
(Sawyers Certified Internal Auditor Reviewer)
A is correct. The relative risk of loss to the company would be lower if a temporary employee were responsible
for the error. This suggests an isolated incident, as the employee will not be able to repeat the error once he or she
has left the purchasing department. However, the error should still be reported to management.
Page 8 of 45

B is incorrect. If employees are not required to consult an approved price list, the company risks buying at too
great a price.
C is incorrect. Even if the company regularly buys from this vendor, there is no assurance that the price is
reasonable unless the purchase order is compared to the approved price list.
D is incorrect. The director should have required the purchase order to be compared to the approved price list
before approving it. Otherwise, the purchase price may exceed the approved price.

A.
B.
C.
D.

10. Which of the following fraudulent entries is most likely to be made to conceal the theft of an asset?
Debit expenses, and credit the asset
Debit the asset, and credit another asset account
Debit revenue, and credit the asset
Debit another asset account, and credit the asset
Answer: A
(Sawyers Certified Internal Auditor Reviewer)
A is correct. Most fraud perpetrators would attempt to conceal their theft by charging it against an expense
account.
B is incorrect. Debiting the stolen asset account would be going in the wrong direction to conceal an asset theft.
C is incorrect. An entry decreasing revenue would be unusual and would stand out.
D is incorrect. This entry would not permanently conceal the fraud. It would simply shift the unreconcilable
balance to another asset account.
11. What computer-assisted audit technique would an auditor use to identify a fictitious or terminated employee?
A. Parallel simulation of payroll calculations.
B. Exception testing for payroll deductions.
C. Recalculations of net pay.
D. Tagging and tracing of payroll tax-rate changes.
Solution: B
a) Incorrect. In a parallel simulation, data that were processed by the engagement clients system are
reprocessed through the auditors program to determine if the output obtained matches the output
generated by the clients system. This technique might identify problems with the clients processing but
would not identify a fictitious or terminated employee.
b) Correct. This type of computer-assisted audit technique (CAAT) program can identify employees
who have no deductions. This is important because fictitious or terminated employees will generally
not have any deductions.
c) Incorrect. A CAAT program can recalculate amounts such as gross pay, net pay, taxes and other
deductions, and accumulated or used leave times. These recalculations can help determine if the payroll
program is operating correctly or if employee files have been altered, but would not identify a fictitious or
terminated employee.
d) Incorrect. In this type of CAAT program, certain actual transactions are tagged, and as they proceed
through the system, a data file is created that traces the processing through the system and permits an
auditor to subsequently review that processing. This would not, however, identify a fictitious or terminated
employee.
12. A chief audit executive (CAE) suspects that several employees have used desktop computers for personal
gain. In conducting an investigation, the primary reason that the CAE chose to engage a forensic information
systems auditor rather than using the organizations information systems auditor is that a forensic information
systems auditor would possess:
A. Knowledge of the computing system that would enable a more comprehensive assessment of the computer
use and abuse.
Page 9 of 45

B. Knowledge of what constitutes evidence acceptable in a court of law.


C. Superior analytical skills that would facilitate the identification of computer abuse.
D. Superior documentation and organization skills that would facilitate in the presentation of findings to
senior management and the board.
Solution: B
a) Incorrect. The organizations information systems auditor would probably have more knowledge of the
organizations computing systems.
b) Correct. The distinguishing characteristic of forensic auditing is the knowledge needed to testify as
an expert witness in a court of law. Although a forensic auditor may possess the other attributes
listed, the organizations information systems auditor may also possess these skills or knowledge
elements.
c) Incorrect. A forensic auditor would not necessarily have analytical or organizational skills that are superior
to those of the organizations auditor.
d) Incorrect. See answer c.
13. An audit committee is concerned that management is not addressing all internal audit observations and
recommendations. What should the audit committee do to address this situation?
A. Require managers to provide detailed action plans with specific dates for addressing audit observations
and recommendations.
B. Require all managers to confirm when they have taken action.
C. Require the chief executive officer to report why action has not been taken.
D. Require the chief audit executive to establish procedures to monitor progress.
Solution: D
a) Incorrect. Management are responsible for ensuring action on all internal audit observations and
recommendations, but some actions may take time to complete and it is not practical to expect that all will
be resolved when an audit committee meets.
b) Incorrect. See answer a.
c) Incorrect. See answer a.
d) Correct. The chief audit executive is responsible for establishing appropriate procedures for
monitoring the progress by management on all internal audit observations and recommendations.
This responsibility should be written into its charter by the audit committee, and progress should be
reported at each audit committee meeting.
14. As part of a manufacturing companys environmental, health, and safety (EHS) self-inspection program,
inspections are conducted by a member of the EHS staff and the operational manager for a given work area or
building. If a deficiency cannot be immediately corrected, the EHS staff member enters it into a tracking
database that is accessible to all departments via a local area network. The EHS manager uses the database to
provide senior management with quarterly activity reports regarding corrective action. During review of the
self-inspection program, an auditor notes that the operational manager enters the closure information and
affirms that corrective action is complete. What change in the control system would compensate for this
potential conflict of interest?
A. No additional control is needed because the quarterly report is reviewed by senior management, providing
adequate oversight in this situation.
B. No additional control is needed because those implementing a corrective action are in the best position to
evaluate the adequacy and completion of that action.
C. After closure is entered into the system, review by the EHS staff member of the original inspection team
should be required in order to verify closure.
D. The EHS department secretary should be responsible for entering all information in the tracking system
based on memos from the operational manager.
Page 10 of 45

Solution: C
a) Incorrect. Although senior management can use the report to question why certain corrective actions may
be behind schedule, they have no way of knowing whether the corrective actions shown as complete were
actually completed.
b) Incorrect. While the operational managers may in fact be the most knowledgeable about the corrective
action, independent verification is preferable.
c) Correct. If there is a step in the process at which someone independent of the area being inspected
can evaluate the adequacy and completeness of corrective action, the potential for closure fraud is
minimized.
d) Incorrect. There is nothing inappropriate about the environmental, health, and safety staff entering the
initial inspection results. Having the secretary enter closure data does not improve controls since there is
still no independent review. It is also less efficient and timely than having the data entered directly in the
field.
15. An audit of an organizations claims department determined that a large number of duplicate payments had
been issued due to problems in the claims processing system. During the exit conference, the vice president of
the claims department informed the auditors that attempts to recover the duplicate payments would be initiated
immediately and that the claims processing system would be enhanced within six months to correct the
problems. Based on this response, the chief audit executive should:
A. Adjust the scope of the next regularly scheduled audit of the claims department to assess controls within
the claims processing system.
B. Monitor the status of corrective action and schedule a follow-up engagement when appropriate.
C. Schedule a follow-up engagement within six months to assess the status of corrective action.
D. Discuss the findings with the audit committee and ask the committee to determine the appropriate followup action.
Solution: B
a) Incorrect. Because the finding is significant, the internal audit activity should not wait until the next
regularly scheduled audit to assess the status of corrective action.
b) Correct. The internal audit activity should monitor the status of the corrective action. A follow-up
engagement should be scheduled when changes to the claims processing system have been
sufficiently completed to allow for testing of adequacy and effectiveness.
c) Incorrect. Although management indicated that the corrections should be completed within six months,
this may not be the case. As a result, the internal audit activity should monitor the status of corrective
action and schedule a follow-up engagement when it is appropriate.
d) Incorrect. Although the findings should be discussed with the audit committee because of their
significance, the scope and timing of a follow-up engagement should be determined by the chief audit
executive based on available information.
16. An organization uses electronic data interchange and on-line systems rather than paper-based documents for
purchase orders, receiving reports, and invoices. Which of the following audit procedures would an auditor
use to determine if invoices are paid only for goods received and at approved prices?
A. Select a statistical sample of major vendors and trace the amounts paid to specific invoices.
B. Use generalized audit software to select a sample of payments and match purchase orders, invoices, and
receiving reports stored on the computer using a common reference.
C. Select a monetary-unit sample of accounts payable and confirm the amounts directly with the vendors.
D. Use generalized audit software to identify all receipts for a particular day and trace the receiving reports to
checks issued.

Page 11 of 45

Solution: B
A. Incorrect. This procedure only provides data on whether payments agree with invoices. It does not provide
data on whether the invoiced amounts are correct.
B. Correct. This would help the auditor determine that all three pieces of data were most likely
matched before payment.
C. Incorrect. As with answer a, this only provides data on whether payments agree with invoices. It does
not provide data on whether the goods were actually received.
D. Incorrect. This provides data only on one day. While it matches items received with those paid, it does not
provide data on whether the billings were correct.
17. A sales department has been giving away expensive items in conjunction with new product sales to stimulate
demand. The promotion seems successful, but management believes the cost may be too high and has asked
for a review by the internal audit activity. Which of the following procedures would be the least useful to
determine the effectiveness of the promotion?
A. Comparing product sales during the promotion period with sales during a similar non-promotion period.
B. Comparing the unit cost of the products sold before and during the promotion period.
C. Performing an analysis of marginal revenue and marginal cost for the promotion period, compared to the
period before the promotion.
D. Performing a review of the sales departments benchmarks used to determine the success of a promotion.
Solution: B
a) Incorrect. This comparison would help highlight the effectiveness of the promotion in increasing sales.
b) Correct. There is no indication that cost of the products sold has changed. The challenge is to
address the effectiveness of the promotion.
c) Incorrect. This is the key analysis as it would show the extent of additional revenue versus cost.
d) Incorrect. This would be helpful because the sales department may have useful information on new
customers and repeat purchases.
18. Which of the following observations by an auditor is most likely to indicate the existence of control
weaknesses over safeguarding of assets?
I.
II.
III.
IV.

a)
b)
c)
d)

A service department's location is not well suited to allow adequate service to other units.
Employees hired for sensitive positions are not subjected to background checks.
Managers do not have access to reports that profile overall performance in relation to other benchmarked
organizations.
Management has not taken corrective action to resolve past engagement observations related to inventory
controls.
I and II only.
I and IV only.
II and III only.
II and IV only.

Solution: D (II and IV only)


I.
II.
III.

Incorrect. This is a symptom of weak controls for achieving organizational goals and objectives, but not
for safeguarding of assets.
Correct. This is a symptom of weak controls for safeguarding of assets.
Incorrect. This is a symptom of weak controls for achieving organizational goals and objectives, but not
for safeguarding of assets.
Page 12 of 45

IV.

Correct. Management's failure to take corrective action on past engagement observations, which
related to safeguarding of assets, is a weakness related to safeguarding of assets.

19. The use of an analytical review to verify the correctness of various operating expenses would not be a
preferred approach if:
A. An auditor notes strong indicators of a specific fraud involving these accounts.
B. Operations are relatively stable and have not changed much over the past year.
C. An auditor would like to identify large, unusual, or non-recurring transactions during the year.
D. Operating expenses vary in relation to other operating expenses, but not in relation to revenue.
Solution: A
a) Correct. If the auditor already suspects fraud, a more directed audit approach would be
appropriate.
b) Incorrect. Relatively stable operating data is a good scenario for using analytical review.
c) Incorrect. Analytical review would be useful in identifying whether large, non-recurring, or unusual
transactions occurred.
d) Incorrect. Analytical review only needs to have accounts related to other accounts or other independent
data. It does not require that they be related to revenue.
20. As part of a preliminary survey of the purchasing function, an auditor read the department's policies and
procedures manual. The auditor concluded that the manual described the processing steps well and contained
an appropriate internal control design. The next engagement objective was to determine the operating
effectiveness of internal controls. Which procedure would be most appropriate in meeting this objective?
a)
b)
c)
d)

Prepare a flowchart.
Prepare a system narrative.
Perform a test of controls.
Perform a substantive test.

Solution: C
a) Incorrect. Flowcharts are most appropriate for studying internal control design. The audit objective is
whether the controls are in place and effective, which indicates the need for a test of controls.
b) Incorrect. System narratives are most appropriate for studying internal control design. The audit objective
is whether the controls are in place and effective, which indicates the need for a test of controls.
c) Correct. Tests of controls, also known as compliance tests, help an auditor determine whether
controls are being followed and are effective. For instance, a policy may require that all large
transactions be approved by a manager. As a test of controls, the auditor may sample large
transactions and review whether manager approval was obtained and whether the proposed
transaction meets all the criteria that the manager was supposed to verify.
d) Incorrect. Substantive tests are tests to determine whether an objective has been achieved and do not
necessarily test internal controls.
Management Accounting Services
1. Product cost under absorption costing is characteristically:
A
.
B.
C.

Higher than under variable costing.


Lower than under variable costing.
Equal to variable costing.
Page 13 of 45

D
.

Higher sometimes and lower sometimes than variable costing.

Solution: A is correct.
2. NFJPIA Co. is considering the following investment proposals. Rank the following proposals using the
profitability index (in order of the most profitable proposal down to the least profitable proposal).
Investment required
Present value of future net cash flows
A
.
B.
C.
D
.

A
160,000
192,000

B
200,000
300,000

C
120,000
168,000

D
150,000
240,000

D, B, C, A
B, D, C, A
B, D, A, C
A, C, B, D

Solution: A is correct.
Profitability index is an investment appraisal technique calculated by dividing the present value of future cash
flows of a project by the initial investment required for the project. Profitability index of the above proposals are
as follows:
Decision Rule. Accept a project if the profitability index is greater than 1, stay indifferent if the profitability index
is zero and don't accept a project if the profitability index is below 1. The ranking of the proposals in terms of
preference using the profitability index is determined as follows.

Present value of future net cash flows


Investment required
Profitability index

A
192,000
160,000
1.2

B
300,000
200,000
1.5

C
168,000
120,000
1.4

D
240,000
150,000
1.6

Page 14 of 45

3. The stock of GVS pays dividends that are expected to grow at a steady rate of 3.2% per year. Investors
expect a rate of return of 11.5% from GVS stock. If the spread between this required rate of return and the
dividend growth rate were to increase by 50 basis points, the percentage change in the stock price would
be:
A
.
B.
C.
D
.

+3.94%
-4.73%
-5.68%
+4.22%

Solution: C is correct.
The dividend discount model (DDM) is a method of valuing a company based on the theory that a stock is worth
the discounted sum of all of its future dividend payments. In other words, it is used to value stocks based on
the net present value of the future dividends.
Using Dividend Discount Model given Po = D1/(k g). When k g = 11.5% - 3.2% = 8.3%, the price is given by
Po = D1/0.083. When the spread increases by 50 basis points and all else stays constant, the price becomes P1=
D1/(0.083 + 0.005) = D1/0.088. The percentage change in the price equals (P1 Po)/Po = (1/0.088
1/0.083)*0.083 = 0.083/0.088 1 = -5.68%. Thus, the stock price falls by 5.68% when the spread between k and
g increases by 50 basis points, all else equal.
4. Which of the following statements is true?
A
.
B.
C.
D
.

A firm with financial leverage has a larger equity multiplier than an otherwise identical firm
with no debt in its capital structure.
The use of debt in a companys capital structure results in tax benefits to the investors who
purchase the companys bonds.
All else equal, a firm with a higher debt ratio will have a lower basic earning power ratio.
All of the statements above are correct.

Solution: A is correct.
A firm with financial leverage means the firm uses debt for financing. Equity multiplier is a measure of financial
leverage, calculated as: Total Assets / Total Stockholders' Equity.
Like all debt management ratios, the equity multiplier is a way of examining how a company uses debt to finance
its assets. It is also known as the financial leverage ratio or leverage ratio. A higher equity multiplier indicates
higher financial leverage, which means the company is relying more on debt to finance its assets.
The use of debt provides tax benefits to the corporations that issue debt, not to the investors who purchase debt (in
the form of bonds).
The basic earning power ratio is calculated as the ratio of Earnings Before Interest and Taxes to Total Assets.
The higher the BEP ratio, the more effective a company is at generating income from its assets. BEP is useful for
comparing firms with different tax situations and different degrees of financial leverage. The basic earning power
ratio would be the same if the only thing that differed between the firms were their debt ratios.
Choice A is correct. Other choices are false.

Page 15 of 45

5. The issuance of serial bonds in exchange for an office building, with the first installment of the bonds due
late this year:
A
.
B.
C.
D
.

Decreases net working capital


Decreases the current ratio
Decreases the quick ratio
Affects all of the answers as indicated

Solution: D is correct
D is correct because the first installment is a current liability which affects the quick ratio, the current ratio, and
working capital.
6. The internal rate of return is the
A
.
B.
C.
D
.

Rate of interest that equates the present value of cash outflows and the present value of cash
inflows
Minimum acceptable rate of return for a proposed investment
Risk-adjusted rate of return
Required rate of return

Solution: A is correct.
7. An analyst has obtained the following information regarding two companies, Company X and Company Y:

Company X and Company Y have the same total assets.


Company X has a higher interest expense than Company Y.
Company X has a lower operating income (EBIT) than Company Y.
Company X and Company Y have the same return on equity (ROE).
Company X and Company Y have the same total assets turnover (TATO).
Company X and Company Y have the same tax rate.

Based on the above information, which of the following statements is true?


A
.
B.
C.
D
.

Company X has a higher times interest earned ratio.


Company X and Company Y has the same debt ratio.
Company X has a higher return on assets.
Company X has a lower profit margin.

Solution: D is correct.
Times interest earned (also called interest coverage ratio) is the ratio of earnings before interest and tax
(EBIT) of a business to its interest expense during a given period. It is a solvency ratio measuring the
ability of a business to pay off its debts.
X has a lower net income considering it has a lower EBIT and higher interest than Y, but the same tax rate
as Y. Sales for each company are the same because they have the same total assets and the same total
assets turnover ratio (TATO = Sales/TA). Therefore, since X has a lower net income and same sales as Y,
it follows that it has a lower profit margin (Net Income /Sales).
Page 16 of 45

8. Which of the following statements relating to breakeven point analysis is correct?


Statement 1
Statement 2
Statement 3
Statement 4
A
.
B.
C.
D
.
E.

Breakeven point is not a good planning tool since the goal of business is to make a
profit.
Focusing on target net income rather than operating income will increase the
breakeven point.
Companies with a greater proportion of fixed costs have a greater risk of loss than
companies with a greater proportion of variable costs.
Breakeven point is the level of sales at which profit is zero.

False, False, True, True


False, True, True, True
True, True, False, False
True, False, False, False
All statements are true.

Solution: A is correct.
Statement 1 is false - breakeven point is an important planning tool that helps managers determine volume
of sales/production needed to be profitable.
Statement 2 is false - the same breakeven point will be calculated whether target operating income or
target net income is used in the CVP calculation.
Statements 3 and 4 are correct.
9. A call option on a share of common stock is more valuable when there is lower
A
.
B.
C.
D
.

Market value of the underlying share


Exercise price on the option
Time to maturity on the option
Variability of market price on the underlying share

Answer: B. Exercise price on the option.


Source: Wiley CPA Exam Review Business Environment and Concepts by Whittington and Delaney (2005)
10. Which of the following will cause a shift in the supply curve of a product?
A
.
B.
C.
D
.

Changes in consumer tastes


Changes in the number of buyers in the market
Changes in production taxes
Changes in the price of the product

Answer: C. Changes in production taxes


A shift in the supply curve may result from:
Page 17 of 45

(1)
(2)
(3)
(4)
(5)
(6)
A
.
B.
C
.
D
.

changes in production technology


changes or expected changes in resource prices
changes in the prices of other goods
changes in taxes or subsidies
changes in the number of sellers in the market
expectations about the future price of the product

Incorrect. Changes in consumer tastes will result in a shift in demand curve.


Incorrect. Changes in the number of buyers in the market will result in a shift in demand curve.
Correct. Changes in production taxes will alter the supply curve.
Incorrect. Change in the price of the product involves movement along the existing supply curve,
not a shift in the supply curve.

Source: Wiley CPA Exam Review Business Environment and Concepts by Whittington and Delaney (2005)
11. Operational budgets are used by a retail company for planning and controlling its business activities. Data
regarding the companys monthly sales for the last 6 months of the year and its projected collection
patterns are shown below.
The cost of merchandise averages 40% of its selling price. The companys policy is to maintain an
inventory equal to 25% of the next months forecasted sales. The inventory balance at cost is Php80,000 as
of June 30.
Forecasted sales
July
August
September
October
November
December

Php775,000
750,000
825,000
800,000
850,000
900,000

What is the budgeted cost of the companys purchase for the month of August?
A.
B.
C.
D.

Php307,500
Php300,000
Php450,500
Php142,500

Answer: A. Php307,500
Sales (Php750,000 40%)
Ending inventory (Php825,000 40% 25%)
Beginning inventory (Php750,000 40% 25%)
Purchases

Php300,000
82,500
(75,000)
Php307,500

Source: Wiley CPA Exam Review Business Environment and Concepts by Whittington and Delaney (2005)
12. The management of ABC Corporation has decided to implement a transfer pricing system. ABCs MIS
department is currently negotiating a transfer price for its services with the four producing divisions of the
company as well as the marketing department. Charges will be assessed based on number of reports
(assume that all reports require the same amount of time and resources to produce). The cost to operate the
Page 18 of 45

MIS department at its full capacity of 1,000 reports per year is budgeted at Php450,000. The user subunits
expect to request 250 reports each this year. The cost of temporary labor and additional facilities used to
produce reports beyond capacity is budgeted at Php480.00 per report. ABC could purchase the same
services from an external Information Services firm for Php700,000. What amounts should be used as the
floor and the ceiling in determining the negotiated transfer price?
Floor
Ceiling
A. Php450
Php500
B. Php456
Php560
C. Php540
Php500
D. Php500
Php540
Answer: B. Floor Php456.00; Ceiling Php560.00
Negotiated transfer prices should fall within a range limited by a ceiling and a floor. The ceiling is the lowest
market price that could be obtained from an external supplier, and the floor equals the outlay costs plus
opportunity cost of the transferring division. Since ABCs MIS department does not have the option to sell
services to external customers, its opportunity cost is Php0. Since all costs of service departments must be covered
by the revenue-producing departments, the MIS departments outlay cost equals its total costs. The departments
full capacity level is 1,000 reports per year. However, the user departments will be requesting 1,250 reports (5 user
subunits 250 reports each). Thus, the MIS department will incur costs of Php120,000 [Php480 (1,250 1,000)]
for the 250 reports above capacity, in addition to the Php450,000 budgeted costs for full capacity. The total cost of
Php570,000 (Php450,000 + Php120,000) is used to calculate the floor. The ceiling is based on the Php700,000 that
would be incurred to purchase MIS services externally. Since the MIS department will be producing 1,250 reports,
the floor is Php456.00 (Php570,000 1,250), and the ceiling is Php560.00 (Php700,000 1,250). At full capacity,
any differential costs of additional production are added to the floor. Php480.00 represents only the differential
cost of producing each report above full capacity, not cost per report for total production. Budgeted costs are based
on production of 1,250 reports, not 1,000.
Source: Wiley CPA Exam Review Business Environment and Concepts by Whittington and Delaney (2005)
13. ABC Company has a single product called JGH. The company normally produces and sells 70,000 JGHs
each year at a selling price of Php45.00 per unit. The companys unit costs at this level of activity are
given below:
Direct materials
Direct labor
Variable manufacturing overhead
Fixed manufacturing overhead
Variable selling expenses
Fixed selling expenses
Total cost per unit

Php10.00
3.50
6.80
8.00
2.00
3.00
Php33.30

Assume that ABC Company has sufficient capacity to produce 100,000 units of JGH each year without
any increase in fixed manufacturing overhead costs. The company could increase its sales by 30% above
the present 70,000 units each year if it were willing to increase the fixed selling expenses by Php100,000.
How much would be the increase in income if the production and sales are increased by 30%?
A.
B.
C.
D.

Php376,700
Php300,000
Php465,600
Php356,750

Answer: A. Php376,700
Selling price

Php45.00
Page 19 of 45

Variable expenses
Direct materials
Direct labor
Variable manufacturing overhead
Variable selling expenses
Unit contribution margin

Php10.00
3.50
6.80
2.00

22.30
Php22.70

Additional contribution (70,000 units 30% Php22.70)


Less: Additional fixed selling costs
Additional income

Php476,700
100,000
Php376,700

Source: Comprehensive Review in MAS (2007) by Bobadilla


14. Combo Music Company produces and sells sensitive precision instruments for industrial use. Four types
of instruments are produced at Canlubang Plant. Data with respect to these four types of products are
given below:
Product Line

Time required to make such unit (in minutes)


Selling price per unit
Variable cost per hour

20
24
108

40
34
48

10
16
72

60
40
20

The Company has 24,000 production hours available after meeting the requirements for the regular
market. The market can absorb all units of any product line that is produced with one exception - only
48,000 units of Product 3 can be sold. In addition, there is a shortage of materials for Product 4.
Materials are available for the production of a total of 12,000 units of Product 4.
What is the number of hours available for Product 2 to maximize profit while observing all the
restrictions?
A 6,000 hours
.
B. 4,000 hours
C. 3,000 hours
D 0 hours
.
Answer: B. 4,000 hours.

Total minutes in an hour


Divided by: Minutes to make each unit
Units produced each hour
Multiplied by: Unit selling price
Sales per hour
Variable cost per hour
Contribution margin per hour

1
60
20
3
24
72
108
(36)

Product Line
2
3
60
60
40
10
1.5
6
34
16
51
96
48
72
3
24

4
60
60
1
40
40
20
20

Page 20 of 45

Product ranking of production and sales to achieve the best product


combination and thus maximize profit:
Product 1
Product 2
Product 3
Product 4

will not be manufactured


3rd
1st
2nd

Allocation of 24,000 production hours:


1
Product 3 (48,000 10/60)
Product 4
Product 2 Remainder
Product 1 None
Total allocated hours

Product Line
2
3
8,000

4
12,000

4,000
0

4,000

8,000

12,000

Source: Reviewer in MAS by Morante


15. Which of the following statements regarding transfer pricing is false?
A
.
B.
C.
D
.

When idle capacity exists, there is no opportunity cost to producing intermediate products
for another division.
Market-based transfer prices should be reduced by any costs avoided by selling internally
rather than externally.
No contribution margin is generated by the transferring division when variable cost-based
transfer prices are used.
The goal of transfer pricing is to provide segment managers with incentive to maximize the
profits of their divisions.

Solution: D is correct
The goal of transfer pricing is to encourage managers to make transfer decisions which maximize profits of the
company as a whole. Some transfers may not be profitable to a particular division, but would effect a cost savings
to the company by avoiding costs of purchasing externally. For example, when a division is already operating at
full capacity and uses variable cost transfer prices, additional production for internal transfer would result in a loss
for the transferring division because no contribution margin is earned to cover the differential fixed costs incurred.
Conversely, internal production may be cheaper to the corporate entity than purchasing the product, in which case
the division should accept the order. However, the division manager is likely to engage in suboptimization by
rejecting the order to enhance the divisions performance, while adversely affecting overall company performance.
Business Law and Taxation
1. Marsh offered to construct the house of Mellow for Php900,000, giving the latter ten (10) days within
which to accept or reject the offer. On the fifth day, before Mellow could make up his mind, Marsh
withdrew his offer. Which of the following is true?
A. The withdrawal of Marshs offer will cause the offer to cease in law, hence, there could be no
concurrence of the offer and the acceptance;
B. In the absence of concurrence of offer and acceptance, there can be no consent;
C. Without consent, there is no perfected contract for the construction of the house of Mellow.
D. All of the above.
Answer: d) All of the above.
Page 21 of 45

Article 1318 of the Civil Code There is no contract unless the following requisites concur:
(1) Consent of the contracting parties;
(2) Object certain which is the subject matter of the contract;
(3) Cause of the obligation which is established.
Article 1319 of the Civil Code Consent is manifested by the meeting of the offer and the acceptance upon the
thing and the cause which are to constitute the contract. The offer must be certain and the acceptance absolute. A
qualified acceptance constitutes a counter offer. Xxx.
2. W, X, and Y are partners. X conveyed his share to Z. Thereafter, the partnership incurred liabilities for
the contract entered into between the partnership and M prior to the sale of Xs share to Z. Which of the
following statements is correct?
A. Z cannot be held liable to M because a partner admitted into a partnership shall become liable for an
obligation only from the time he took part in the control of the business.
B. Z cannot be held liable to M because as a partner, Z shall be subject to all the restrictions and
liabilities of a partnership.
C. Z can be held liable pro-rata with the other partners regardless of the time the liability was incurred.
D. Z can be held liable to M, but his liability shall be satisfied only out of partnership property, unless
there is a stipulation to the contrary.
Answer: D Z can be held liable to M, but his liability shall be satisfied only out of partnership property,
unless there is a stipulation to the contrary.
Article 1826 of the Civil Code provides: A person admitted as a partner into an existing partnership is liable for
all the obligations of the partnership arising before his admission as though he had been a partner when such
obligations were incurred, except that his liability shall be satisfied only out of partnership property, unless there is
a stipulation to the contrary.
3. ABC owns 15% of the subscribed capital stock of XYZ corporation which owns an office building. ABC
owns a security and janitorial agency. ABC, a director of XYZ Corporation, engaged the services of his
own company to perform services to the office building. Which of the following conditions is NOT
required in order for the service contract between ABC and XYZ to be valid?
A. The service contract must be approved by the board of directors of XYZ corporation;
B. ABCs vote was not necessary to constitute a quorum at the meeting of the board of directors;
C. ABCs vote was not necessary for the approval of the service contracts.
D. The service contract must be ratified by all the stockholders of the corporation.
Answer: D The service contract must be ratified by all the stockholders of the corporation.
Section 32 of the Corporation Code provides: A contract of the corporation with one or more of its directors or
trustees or officers is voidable, at the option of such corporation, unless all the following conditions are present:
1. That the presence of such director of trustee in the board meeting in which the contract was approved was
not necessary to constitute a quorum for such meeting;
2. That the vote of such director or trustee was not necessary for the approval of the contract;
3. That the contract is fair and reasonable under the circumstances; and
4. That in the case of an officer, the contract has been previously authorized by the board of directors.
Where the first two conditions set forth in preceding paragraph is absent, in case of a contract with a director or
trustee, such contract may be ratified by the vote of the stockholders representing at least two-thirds of the
outstanding capital stock or of at least two thirds of the members in a meeting called for the purpose: Provided,
That full disclosure of the adverse interest of the directors or trustees involved is made at such meeting: Provided,
however, That the contract is fair and reasonable under the circumstances.
4. In line with the Trust Fund Doctrine that generally renders it unlawful for the corporation to return assets
to the stockholders representing capital, under what conditions may a stock corporation acquire its own
shares:
A. There exists in the corporate books unrestricted retained earnings to cover the repurchase of shares;
Page 22 of 45

B. The repurchase of shares must be for a legitimate business purpose;


C. Both A and B.
D. Neither A nor B.
Answer: C Both A and B.
Section 41 of the Corporation Code provides: A stock corporation shall have the power to purchase or acquire its
own shares for a legitimate corporate purposes or purposes, including but not limited to the following cases:
Provided, That the corporation has unrestricted retained earnings in its books to cover the shares to be purchased
or acquired:
a. To eliminate fractional shares arising out of stock dividends;
b. To collect or compromise an indebtedness to the corporation, arising out of unpaid subscription, in a
delinquency sale, and to purchase delinquent shares sold during said sale; and
c. To pay dissenting or withdrawing stockholders entitled to payment for their shares under the provisions of this
Code.
5. In a stockholders meeting, Mr. Bean dissented from the corporate act converting preferred voting shares
to non-voting shares. Mr. Bean submitted his certificates of stock for notation that his shares are
dissenting. The next day, Mr. Bean transferred his shares to Ms. Dora to whom new certificates were
issued. Which of these statements is correct?
A. Mr. Bean can exercise the right of appraisal because he dissented from the corporate act.
B. Mr. Bean can no longer exercise the right of appraisal because he already transferred his shares to Ms.
Dora.
C. Ms. Dora should exercise the right of appraisal on behalf of Mr. Bean.
D. None of the statements is correct.
Answer: B. Mr. Bean can no longer exercise the right of appraisal because he already transferred his shares
to Ms. Dora.
Section 86 of the Corporation Code provides: Within ten (10) days after demanding payment for his shares, a
dissenting stockholder shall submit the certificates of stock representing his shares to the corporation for notation
thereon that such shares are dissenting shares. His failure to do so shall, at the option of the corporation, terminate
his rights under this Title. If shares represented by the certificates bearing such notation are transferred, and the
certificates consequently cancelled, the rights of the transferor as a dissenting stockholder under this Title shall
cease and the transferee shall have all the rights of a regular stockholder; and all dividend contributions which
would have accrued on such shares shall be paid to the transferee.
6. ABC Corporation has 10 members in the Board of Directors. During the election of officers, one of its
members got sick, while the other two went abroad, thus, the three of them failed to attend the meeting
and cast their vote. Nonetheless, A acquired four votes for the position of Secretary. Is A validly elected
as an officer?
A. No. The Code requires the vote of majority of all the members of the Board of Directors.
B. Yes. The Code requires only the decision of at least a majority of directors present at the election.
C. Yes. The articles of incorporation or by-laws may provide for lesser majority in case of election of
officers.
D. No. The articles of incorporation require the quorum.
Answer: A. No. The Code requires the vote of majority of all the members of the Board of Directors.
Section 25 of the Corporation Code: Immediately after their election, the directors of a corporation must
formally organize by the election of a president, who shall be a director, a treasurer who may or may not be a
director, a secretary who shall be a resident and citizen of the Philippines, and such other officers as may be
provided for in the by-laws. Any two or more positions may be held concurrently by the same person, except that
no one shall act as president and secretary or as president and treasurer at the same time.

Page 23 of 45

The directors or trustees and officers to be elected shall perform the duties enjoined on them by law and by the bylaws of the corporation. Unless the articles of incorporation or the by-laws provide for a greater majority, a
majority of the number of directors or trustees as fixed in the articles of incorporation shall constitute a quorum for
the transaction of corporate business, and every decision of at least a majority of the directors or trustees present at
a meeting at which there is a quorum shall be valid as a corporate act, except for the election of officers which
shall require the vote of a majority of all the members of the board.
Directors or trustees cannot attend or vote by proxy at board meetings.
7. A, B, C, D, and E entered into a contract of partnership to organize events such as weddings and birthday
celebrations. A and B are the only limited partners. During one of the parties that the partnership
organized, D totally forgot her task to turn on the lights during the entrance of the debutant. As a result,
the debutant slipped off injuring her ankle. The victim sues the partnership for damages. Choose the best
answer.
A. A and B are not liable because they are limited partners.
B. The partnership is liable for damages to the same extent as D.
C. Only the general partners are liable in equal proportion.
D. Only D is liable.
Answer: B. The partnership is liable for damages to the same extent as D.
Article 1822 of the Civil Code provides: Where, by any wrongful act or omission of any partner acting in the
ordinary course of the business of the partnership or with the authority of his co-partners, loss or injury is caused
to any person, not being a partner in the partnership, or any penalty is incurred, the partnership is liable therefor to
the same extent as the partner so acting or omitting to act.
8. X owes Y Php100,000 payable in one year, at 10% interest per annum. On maturity, X pays Y Php50,000
which Y acknowledged having received without mentioning the payment of interest. Which of the
following is correct?
A. Principal has already been paid.
B. Interest is waived.
C. Payment will be applied first to the interest.
D. Obligation is extinguished.
Answer: C. Payment will be applied first to the interest.
Article 1253 of the Civil Code provides: If the debt produces interest, payment of the principal shall not
be deemed to have been made until the interests have been covered.
9. ABC Corporation is a stock corporation whose principal place of business is in Quezon City. If a regular
meeting of its stockholders shall be held, one of the requirements is:
A. Notice of meetings must be published in a newspaper of general circulation two weeks before the
scheduled meeting.
B. Quorum must consist of all the stockholders.
C. Written notice of meetings shall be sent at least two weeks prior to the meeting, unless a different
period is required by the by-laws.
D. All of the above.
Answer: C. Written notice of meetings shall be sent at least two weeks prior to the meeting, unless a
different period is required by the by-laws.
Section 50 of the Corporation Code provides: Regular meetings of stockholders or members shall be held
annually on a date fixed in the by-laws, or if not so fixed, on any date in April of every year as determined by the
board of directors or trustees: Provided, That written notice of regular meetings shall be sent to all stockholders or
members of record at least two (2) weeks prior to the meeting unless a different period is required by the by-laws.
Page 24 of 45

Special meetings of stockholders or members shall be held at any time deemed necessary or as provided in the bylaws; Provided, however, That at least one (1) week written notice shall be sent to all stockholders or members,
unless otherwise provided in the by-laws. Xxx.
10. XYZ Corporations by-laws provide that A, one of its directors, shall receive 10% of the net income
before tax of the corporation as his yearly compensation. However, B, the Corporations president,
contends that such provision directly violates the Code prohibiting the compensation of directors. Are
directors absolutely not entitled to compensation?
A. Yes. The Code expressly provides that directors shall not receive any compensation, except for
reasonable per diems.
B. No. Directors are entitled to compensation when authorized by the by-laws or by vote of stockholder
representing at least majority of the outstanding capital stock, subject to limitations that the total
yearly compensation must exceed 10% of the net income before income tax of the compensation
during the preceding year.
C. No. Directors are entitled when authorized by the by-laws or by vote of stockholders representing at
least majority of the outstanding capital stock, subject to limitations that the total yearly compensation
shall not exceed 10% of the net income before tax of the corporation during the preceding year.
D. Yes. Directors are not entitled to compensation.
Answer: C. No. Directors are entitled when authorized by the by-laws or by vote of stockholders
representing at least majority of the outstanding capital stock, subject to limitations that the total yearly
compensation shall not exceed 10% of the net income before tax of the corporation during the preceding
year.
Section 30 of the Corporation Code provides: Compensation of directors. -- In the absence of any provision in
the by-laws fixing their compensation, the directors shall not receive any compensation, as such directors, except
for reasonable per diems: Provided, however, That any such compensation other than per diems may be granted to
directors by the vote of the stockholders representing at least a majority of the outstanding capital stock at a
regular or special stockholders meeting. In no case shall the total yearly compensation of directors, as such
directors, exceed ten percent (10%) of the net income before income tax of the corporation during the preceding
year.
11. Employee J, a resident Filipino citizen, is a teacher in Monsters University. He earns a monthly salary of
P25,000. For the year 2012, he also received a discretionary bonus amounting to P40,000. His wife, K,
gave birth to their fourth child. The names of their children and the birthdays are as follows:
Ethel
Luz
Edmon
d
Grace

3/11/199
1
6/14/199
8
6/19/200
6
9/15/201
2

For the year ended 2012, how much basic personal exemption can Employee J claim?
A.
B.
C.
D.
E.

P150,000
P125,000
P100,000
P50,000
Zero

Answer: D. P50,000
Page 25 of 45

12. Co. C had the following data in the month of September:


Sales
Purchases of goods
Purchase of machines
Machine Life

P1,900,000
1,260,000
1,440,000
8 years

Assuming that the said transactions are properly substantiated for VAT purposes, compute the VAT
payable. Note: The amounts are exclusive of VAT.
A. Php70,920
B. Php76,800
C. Php73,920
D. Php225,120
Answer: C. P73,920
228,000

Output VAT (P1,900,000 x 12%)


Input VAT
Purchase of goods (P1,260,000 x 12%)
Purchase of machines* (P1,440,000 x 12%/60 months)
Net VAT Payable

151,200
2,880

154,080
73,920

13. On July 31, 2013, Esperanza received a preliminary assessment notice from the BIR demanding that she
pays P180,000.00 deficiency income taxes on her 2011 income. How many days from July 31, 2013
should Esperanza respond to the notice?
A. 180 days
B. 30 days
C. 60 days
D. 15 days
Answer: D. 15 days
14. The gross estate of Ms. Layda amounted to P5 million. The actual funeral expenses are as follows:
Mourning apparel of the surviving spouse
Mourning apparel of married children
Publication charges for death notices
Interment and/or cremation fees and charges
Total

P100,000
90,000
70,000
50,000
310,000

How much is the total deductible funeral expense?


A. P250,000
B. P200,000
C. P270,000
D. P180,000
Answer: B. P200,000
Actual funeral expenses
Mourning apparel of the surviving spouse
Publication charges for death notices
Interment and/or cremation fees and charges

100,000
70,000
50,000
220,000
Page 26 of 45

5% of gross estate

250,000

Limit

200,000

15. Gianna Corporation is engaged in the business of publishing magazines as well as the merchandising of
office supplies. For CY 2012, it has the following revenue/sales:
Revenue from magazines
Sales from office supplies

587,400
197,500

Further, Gianna incurred entertainment, amusement and recreation expenses (EAR) amounting to P8,350
and 2/3 of which was incurred for the publishing of magazines. How much EAR is deductible for income
tax purposes?
A.
B.
C.
D.

P8,657.33
P8,350.00
P6,861.50
P6,554.17

Answer: D. P6,554.17
Revenue

Actual EAR

Revenue from
magazines

587,400

2/
3

5,56
6.67

Sales from office


supplies

197,500

1/
3

2,78
3.33
8,35
0.00

EAR LIMIT
(1% of
5,874.0 revenue from
0 magazines)
(0.5% of sales
987.5 from office
0 supplies)

Deductible EAR
5,566.6
7
987.5
0
6,554.1
7

Theory of Accounts
1. This refers to IASBs main vehicle for consulting the public and sets out a specific proposal in the form of
a proposed IFRS (or amendment to an IFRS).
A. Final Standards
B. Due Process Handbook
C. Amendments
D. Exposure Draft
Answer: D. Exposure Draft
Due Process Handbook for the IASB and IFRS Interpretations Committee (February 2013) provides:
An Exposure Draft is the IASBs main vehicle for consulting the public and therefore includes an invitation to
comment, setting out the issues that the IASB has identified as being of particular interest. Although it is
normally included with the ballot draft, it is not necessary for the IASB to ballot the invitation to comment.
(Par. 6.3)
An Exposure Draft sets out a specific proposal in the form of a proposed Standard (or amendment to a
Standard) and is therefore generally set out in the same way as, and has all of the components of, a Standard.
(Par. 6.2)
Page 27 of 45

2. Which of the following is not an indicator of situations that would individually or in combination lead to a
lease being classified as a finance lease?
A
.
B.
C.
D
.

The leased assets are of such a specialized nature that only the lessee can use them without
major modifications.
Gains or losses from the fluctuation in the fair value of the residual accrue to the lessor.
The lessee has the ability to continue the lease for a secondary period at a rent that is
substantially lower than market rent.
If the lessee can cancel the lease, the lessor's losses associated with the cancellation are
borne by the lessee.

Answer: B. Gainsor losses from the fluctuation in the fair value of the residual accrue to the lessor.
Paragraphs 10-11 of PAS 17, Leases state:
Whether a lease is a finance lease or an operating lease depends on the substance of the transaction rather
than the form of the contract.1 Examples of situations that individually or in combination would normally
lead to a lease being classified as a finance lease are:
(a) the lease transfers ownership of the asset to the lessee by the end of the lease term;
(b) the lessee has the option to purchase the asset at a price that is expected to be sufficiently lower than the
fair value at the date the option becomes exercisable for it to be reasonably certain, at the inception of
the lease, that the option will be exercised;
(c) the lease term is for the major part of the economic life of the asset even if title is not transferred;
(d) at the inception of the lease the present value of the minimum lease payments amounts to at least
substantially all of the fair value of the leased asset; and
(e) the leased assets are of such a specialized nature that only the lessee can use them without major
modifications.
Indicators of situations that individually or in combination could also lead to a lease being classified as a
finance lease are:
(a) if the lessee can cancel the lease, the lessor's losses associated with the cancellation are borne by the
lessee;
(b) gains or losses from the fluctuation in the fair value of the residual accrue to the lessee (for example,
in the form of a rent rebate equaling most of the sales proceeds at the end of the lease); and
(c) the lessee has the ability to continue the lease for a secondary period at a rent that is substantially
lower than market rent.
3. Which of the following statement(s) is/are incorrect?
Statement I: PAS 17, Leases, permits initial direct costs incurred by lessors in negotiating an operating
lease to be charged as expenses when incurred.
Statement II: It is mandatory to apply the finance lease accounting methodology set out in PAS 17 to
investment properties held under operating leases.
A
.
B.
C.
D
.

None of the above


Statement I only
Statement II only
Statements I and II

Answer: B. Statement I only


Paragraph 52 of PAS 17 provides:
Page 28 of 45

Initial direct costs incurred by lessors in negotiating and arranging an operating lease shall be added to the
carrying amount of the leased asset and recognized as an expense over the lease term on the same basis as
the lease income.
Paragraph IN5 of PAS 17 provides:
Although PAS 40, Investment Property prescribes the measurement models that can be applied to investment
properties held, it requires the finance lease accounting methodology set out in this Standard to be used for
investment properties held under leases.
Paragraph 25 of PAS 40 provides:
The initial cost of a property interest held under a lease and classified as an investment property shall be as
prescribed for a finance lease by paragraph 20 of PAS 17, i.e., the asset shall be recognized at the lower of
the fair value of the property and the present value of the minimum lease payments. An equivalent amount
shall be recognized as a liability in accordance with that same paragraph.
4. Which of the following statement(s) regarding impairment testing under PAS 36, Impairment of Assets,
is/are correct?
Statement I: Value in use is the undiscounted future cash flows expected to be derived from an asset or
cash-generating unit.
Statement II: When the amount estimated for an impairment loss is greater than the carrying amount of
the asset to which it relates, an entity shall automatically recognize a liability.
Statement III: If it is not possible to estimate the recoverable amount of an individual asset, the entity
should determine the recoverable amount of the cash generating unit to which the asset belongs.
Statement IV: PAS 36 requires an intangible asset with an indefinite useful life to be tested for
impairment annually by comparing its carrying amount with its recoverable amount, irrespective of
whether there is any indication that it may be impaired. However, the most recent detailed calculation of
such an asset's recoverable amount made in a preceding period may be used in the impairment test for that
asset in the current period as long as certain criteria provided by PAS 36 are met.
A
.
B.
C.
D
.

Statements III and IV


Statements II, III and IV
Statements I and III
Statements I, III and IV

Answer: A. Statements III and IV


Par. 6 of PAS 36, Impairment of Assets, provides:
Value in use is the present value of the future cash flows expected to be derived from an asset or cashgenerating unit.
Par. 62 of PAS 36, Impairment of Assets, provides:
When the amount estimated for an impairment loss is greater than the carrying amount of the asset to which
it relates, an entity shall recognize a liability if, and only if, that is required by another Standard.
Par. 22 of PAS 36, Impairment of Assets, provides:
Recoverable amount is determined for an individual asset, unless the asset does not generate cash inflows
that are largely independent of those from other assets or groups of assets. If this is the case, recoverable
amount is determined for the cash-generating unit to which the asset belongs
Par. 24 of PAS 36, Impairment of Assets, provides:
Page 29 of 45

Paragraph 10 requires an intangible asset with an indefinite useful life to be tested for impairment annually
by comparing its carrying amount with its recoverable amount, irrespective of whether there is any indication
that it may be impaired. However, the most recent detailed calculation of such an asset's recoverable amount
made in a preceding period may be used in the impairment test for that asset in the current period, provided
all of the following criteria are met:
(a) if the intangible asset does not generate cash inflows from continuing use that are largely independent of
those from other assets or groups of assets and is therefore tested for impairment as part of the cashgenerating unit to which it belongs, the assets and liabilities making up that unit have not changed
significantly since the most recent recoverable amount calculation;
(b) the most recent recoverable amount calculation resulted in an amount that exceeded the asset's carrying
amount by a substantial margin; and
(c) based on an analysis of events that have occurred and circumstances that have changed since the most
recent recoverable amount calculation, the likelihood that a current recoverable amount determination
would be less than the asset's carrying amount is remote.
5. Which of the following are considered adjusting events after the reporting period in accordance with PAS
10, Events after the Reporting Period?
I.
II.
III.
IV.
V.
A
.
B.
C.
D
.

Bankruptcy of a customer that occurs after the reporting period


Decline in market value of investments
Declaration of dividends to holders of equity instruments after the reporting period
Determination after the reporting period of the amount of profit-sharing or bonus payments, if
the entity had a present legal or constructive obligation at the end of the reporting period to
make such payments as a result of events before that date
Abnormally large change in asset prices or foreign exchange rates

Statements I, IV and V
Statements I, II and IV
Statements I and IV
Statements I, II, IV and V

Answer: C. Statements I and IV


Paragraph 9 of PAS 10, Events after the Reporting Period provides the following examples of adjusting
events after the reporting period as follows:
(a) the settlement after the reporting period of a court case that confirms that the entity had a present
obligation at the end of the reporting period. The entity adjusts any previously recognized provision related to
this court case in accordance with PAS 37, Provisions, Contingent Liabilities and Contingent Assets or
recognizes a new provision. The entity does not merely disclose a contingent liability because the settlement
provides additional evidence that would be considered in accordance with paragraph 16 of PAS 37.
(b) the receipt of information after the reporting period indicating that an asset was impaired at the end of
the reporting period, or that the amount of a previously recognized impairment loss for that asset needs to be
adjusted. For example:
(i)

the bankruptcy of a customer that occurs after the reporting period usually confirms that a loss
existed at the end of the reporting period on a trade receivable and that the entity needs to adjust
the carrying amount of the trade receivable; and

(ii)

the sale of inventories after the reporting period may give evidence about their net realizable
value at the end of the reporting period.

(c) the determination after the reporting period of the cost of assets purchased, or the proceeds from assets
sold, before the end of the reporting period.
Page 30 of 45

(d) the determination after the reporting period of the amount of profit-sharing or bonus payments, if the
entity had a present legal or constructive obligation at the end of the reporting period to make such payments
as a result of events before that date (see PAS 19, Employee Benefits).
(e)

the discovery of fraud or errors that show that the financial statements are incorrect.

6. When disclosing financial instruments, an entity shall group them into classes as follows:
I.
II.
III
.
A
.
B.
C.
D

Group the financial instruments into classes that are appropriate to the nature of the information
disclosed and that take into account the characteristics of those financial instruments.
Group the financial instruments according to the categories of financial instruments in PAS 39,
Financial Instruments: Recognition and Measurement.
Provide sufficient information to allow reconciliation to the line items presented in the statement
of financial position.
I only.
I and II.
I and III
I, II and III.

Answer C. I and III are true:


A
.

B.

C
.

Incorrect. Although Statement I is correct such that PFRS 7 paragraph 6 provides that an entity
shall group financial instruments into classes that are appropriate to the nature of the information
disclosed and that take into account the characteristics of those financial instruments, the standard
also requires that sufficient information shall be provided to permit reconciliation to the line items
presented in the statement of financial position (Statement III).
Incorrect. Although Statement I is correct such that PFRS 7 paragraph 6 provides that an entity
shall group financial instruments into classes that are appropriate to the nature of the information
disclosed and that take into account the characteristics of those financial instruments, grouping the
financial instruments into categories (Statement II) is not consistent with the standards requirement
to group financial instruments into classes.
Correct. I and III are true. PFRS 7.6 provides that an entity shall group financial instruments
into classes that are appropriate to the nature of the information disclosed and that take into
account the characteristics of those financial instruments. Also, the standard requires that
sufficient information shall be provided to permit reconciliation to the line items presented in
the statement of financial position.
Incorrect. Statements I and III are consistent with the requirements in PFRS 7.6 about grouping
financial instruments into classes. However, grouping the financial instruments into categories
(Statement II) is not consistent with the standards requirement to group financial instruments into
classes. The correct answer is C.
PFRS 7.6 provides when this PFRS requires disclosures by class of financial instrument, an entity shall
group financial instruments into classes that are appropriate to the nature of the information disclosed and
that take into account the characteristics of those financial instruments. An entity shall provide sufficient
information to permit reconciliation to the line items presented in the statement of financial position.

7. Which of the following statement(s) is/are true?

Page 31 of 45

Statement I:

If an entity's interim financial report is in compliance with PAS 34, Interim


Financial Reporting, that fact shall be disclosed. An interim financial report shall
not be described as complying with PFRSs unless it complies with some or
substantially all of the requirements of PFRSs.
If an entity publishes a condensed set of financial statements in its interim
financial report, the form and content of those statements shall always conform to
the requirements of PAS 1, Presentation of Financial Statements.
Costs that are incurred unevenly during an entity's financial year shall be
anticipated or deferred for interim reporting purposes if, and only if, it is also
appropriate to anticipate or defer that type of cost at the end of the financial year.

Statement II:
Statement III:

A
.
B.
C.
D
.

Statement III only.


Statements I and II.
Statements II and III
Statements I, II and III.

Answer: A. Statement III only.


PAS 34 provides:
If an entity's interim financial report is in compliance with this Standard, that fact shall be disclosed. An
interim financial report shall not be described as complying with PFRSs unless it complies with all of the
requirements of PFRSs (Par.19)
If an entity publishes a complete set of financial statements in its interim financial report, the form and
content of those statements shall conform to the requirements of PAS 1 for a complete set of financial
statements. (Par. 9)
Costs that are incurred unevenly during an entity's financial year shall be anticipated or deferred for
interim reporting purposes if, and only if, it is also appropriate to anticipate or defer that type of cost at the
end of the financial year. (Par. 39)
8. Which of the following statement(s) is/are incorrect in accordance with PAS 28, Investment in Associates
and Joint Ventures?
Statement I:
Statement II:

A
.
B.
C.
D
.

If an investment in an associate becomes an investment in a joint venture, the entity


continues to apply the equity method and does not remeasure the retained interest.
When downstream transactions provide evidence of a reduction in the net realizable
value of the assets to be sold or contributed, or of an impairment loss of those assets,
those losses shall be recognized to the extent of the investors interest in the associate.

All of the above.


Statement I only
Statement II only
None of the above.

Answer: C. Statement II only


PAS 28 states that:

Page 32 of 45

Paragraph 24 - If an investment in an associate becomes an investment in a joint venture or an investment


in a joint venture becomes an investment in an associate, the entity continues to apply the equity method and
does not remeasure the retained interest.
Paragraph 29 - When downstream transactions provide evidence of a reduction in the net realizable value
of the assets to be sold or contributed, or of an impairment loss of those assets, those losses shall be
recognized in full by the investor
9. Under the PAS 19, Employee Benefits (Revised), the following are components of defined benefit costs
under defined benefit plans, except:
A
.
B.
C.
D
.

Service cost in profit or loss


Net interest on the net defined benefit liability in profit or loss
Remeasurements of the net defined benefit liability (asset) in other comprehensive income
Remeasurements of the net defined benefit liability (asset) in profit or loss

Answer: D. Remeasurements of the net defined benefit liability (asset) in profit or loss
Par. 120 of PAS19R states:
An entity shall recognize the components of defined benefit cost, except to the extent that another PFRS
requires or permits their inclusion in the cost of an asset, as follows:
(a) service cost (see paragraphs 66-112) in profit or loss;
(b) net interest on the net defined benefit liability (asset) (see paragraphs 123-126) in profit or loss; and
(c) remeasurements of the net defined benefit liability (asset) (see paragraphs 127-130) in other
comprehensive income.
10. Which of the following is/are an example of a situation in which offsetting is inappropriate?
Statement I: Financial assets and financial liabilities that arise from financial instruments having the
same primary risk exposure (e.g., assets and liabilities within a portfolio of forward contracts or other
derivative instruments) but involve different counterparties.
Statement II: Financial or other assets pledged as collateral for non-recourse financial liabilities
Statement III: Obligations incurred as a result of events giving rise to losses are expected to be
recovered from a third party by virtue of a claim made under an insurance contract.
Statement IV: The entity currently has a legally enforceable right to set-off the recognized amounts and
intends either to settle on a net basis, or to realize the asset and settle the liability simultaneously.
A
.
B.
C.
D
.

Statement I only
Statements I and II only
Statements I, II and II I only
Statements I, II, III and IV

Answer: C. Statements I, II and III only


Paragraph 49 of PAS 32, Financial Instruments: Presentation, states that the conditions set out in paragraph
42 of PAS 32 are generally not satisfied and offsetting is usually inappropriate when:
(a) several different financial instruments are used to emulate the features of a single financial instrument
(a 'synthetic instrument');
Page 33 of 45

(b) financial assets and financial liabilities arise from financial instruments having the same primary risk
exposure (for example, assets and liabilities within a portfolio of forward contracts or other derivative
instruments) but involve different counterparties;
(c) financial or other assets are pledged as collateral for non-recourse financial liabilities;
(d) financial assets are set aside in trust by a debtor for the purpose of discharging an obligation without
those assets having been accepted by the creditor in settlement of the obligation (for example, a sinking fund
arrangement); or
(e) obligations incurred as a result of events giving rise to losses are expected to be recovered from a third
party by virtue of a claim made under an insurance contract.
Practical Accounting 1
1. WXY Company has an overdue 8% note payable to First Bank at Php8,000,000 and accrued interest of
Php640,000. As a result of a restructuring agreement entered on December 31, 2014, First Bank agreed
to the following:
a.
b.
c.
d.

The principal obligation is reduced to Php7,000,000.


The accrued interest of Php640,000 will be waived.
The date of maturity is extended to December 31, 2018.
Annual interest of 10% is to be paid in 4 years every December 31 starting in 2015.

The present value of 1 at 8% for 4 periods is 0.735 and the present value of an ordinary annuity of 1 at 8%
for 4 periods is 3.31. How much is the gain on extinguishment to be recognized for 2014 assuming that
the prevailing market rate for similar note is at 8%?
A.
B.
C.
D.

P1,178,000
P538,000
P1,641400
P0

Solution: A is correct
Original principal
Accrued interest
Carrying amount of overdue debt and interest
Restructured principal @ present value
(Php7,000,000 0.735)
Restructured interest @ present value
(Php7,000,000 10% 3.31)
Gain on extinguishment (13.6% of the carrying
amount of overdue debt and interest)

Php8,000,000
640,000
Php8,640,000
5,145,000
2,317,000

7,462,000
Php 1,178,000

Par. 40 of PAS 39, Financial Instruments: Recognition and Measurement, provides:


a substantial modification of the terms of an existing financial liability or a part of it (whether or not
attributable to the financial difficulty of the debtor) shall be accounted for as an extinguishment of the
original financial liability and the recognition of a new financial liability.
Par. 41 of PAS 39 provides:
The difference between the carrying amount of a financial liability (or part of a financial liability)
extinguished or transferred to another party and the consideration paid, including any non-cash assets
transferred or liabilities assumed, shall be recognized in profit or loss.
Page 34 of 45

Par. 62 of Application Guidance of PAS 39 provides:


For the purpose of paragraph 40, the terms are substantially different if the discounted present value of
the cash flows under the new terms, including any fees paid net of any fees received and discounted using
the original effective interest rate, is at least 10 per cent different from the discounted present value of the
remaining cash flows of the original financial liability. If an exchange of debt instruments or modification
of terms is accounted for as an extinguishment, any costs or fees incurred are recognized as part of the
gain or loss on the extinguishment. If the exchange or modification is not accounted for as an
extinguishment, any costs or fees incurred adjust the carrying amount of the liability and are amortized
over the remaining term of the modified liability.
2. ABC Company has five million ordinary shares in issue at the beginning of Year 1. In the middle of Year 2
it announces a rights issue whereby all existing shareholders will be entitled to buy one share for every
five they hold, at a price of 50. Immediately prior to the issue, the share price was 80. The profits for Years
1, 2, and 3 were 200 million, 300 million, and 360 million, respectively. The rights were exercised
immediately upon issuance. What are the (restated) basic earnings per share for each of these three years?
A
.
B.
C.
D
.

Year 1: 40.00, Year 2: 52.93, Year 3: 60.00


Year 1: 40.00, Year 2: 54.55, Year 3: 60.00
Year 1: 37.49, Year 2: 52.93, Year 3: 60.00
Year 1: 37.49, Year 2: 54.55, Year 3: 60.00

Solution: C is correct
EPS as originally calculated in Year 1 will be 40 (200 million / 5 million).
In the following year, it is necessary to calculate the bonus element of the rights issue. This is done as follows:
Computing the theoretical ex-rights value per share
Fair value of all outstanding shares before exercise of rights + Total amount received from exercise of rights

Number of shares outstanding before exercise of rights + Number of shares issued in exercise of rights
400,000,000 + 50,000,000
=

= 75

5,000,000 + 1,000,000

Computing the adjustment factor


Fair value per share prior to exercise of rights

= 80 / 75 = 1.067

Theoretical ex-rights value per share

Computing basic earnings per share


Page 35 of 45

Year 1 as originally reported 40.00


Year 1 as restated (200,000,000 / [5,000,000 x 1.067]) = 37.49
Year 2
300,000,000

52.93

(5,000,000 x 1.067 x 0.5) + (6,000,000 x 0.5)


Year 3
360,000,000

60.00

6,000,000
3. LKF Company has the following outstanding liabilities and capital as of December 31, 2012:
a. Three million ordinary shares issued at P1.00 (which is also the par value) three years ago. LKF
Company has bought back 2,500 of its shares this year.
b. P6,500,000 million loan with GHC Bank maturing August 31, 2013. The company has communicated
with GHC Bank that it will roll over the said loan on November 7, 2012. Upon rollover, the maturity of
the said loan is deferred to August 31, 2014. This was noted by the bank.
c. Accounts payable amounting to P3,489,000. This account does not include the following items:
(1) Purchases from suppliers amounting to P678,000 shipped FOB shipping point. The goods were in
transit as of December 31, 2012.
(2) Purchases from suppliers amounting to P58,000 shipped FOB destination. The goods were in transit
as of December 31, 2012.
d. Dividends on cumulative preference shares amounting to P3,500,000. The declaration of the said
dividends was made only on February 6, 2013.
e. P2,890,000 purchase commitment entered during 2012 to buy 57,800 kilos of corn to be delivered next
year.
f. P7,590,000 million loan with JHZ Bank maturing September 30, 2013. The company has a credit line
with KLM Bank and plans to drawdown Php7,590,000 million from KLM Bank on September 30,
2013 to pay off the loan with JHZ Bank. The new loan will mature on September 30, 2016.
g. Deferred tax liabilities as of December 31, 2012 amounted to Php5,673,000.
h. P9,000,000 convertible debt with VCD Company maturing June 30, 2013. The Company may either
settle the debt through payment or issuance of ordinary shares.
i. P1,000,000,000 redeemable preference shares. The preference shares were issued five years ago. The
company is required to redeem the shares on or before May 31, 2013.
How much is the current liability to be presented in the balance sheet as of December 31, 2012?
A
.
B.
C.
D
.

P1,023,647,000
P1,011,757,000
P1,014,647,000
P1,020,757,000

Solution: D is correct
c.

Php3,489,000
Page 36 of 45

c.1
f.
h.
i.
Total current liabilities

678,000
7,590,000
9,000,000
1,000,000,000
Php1,020,757,000

4. On January 1, 2014, KGY Company contracted with a contractor to construct an equipment for
P43,000,000. The equipment was completed during the year. KGY Company is required to make the
following payments in 2012:
January 1
March 1
July 1
September 1
December 1
Total

P4,300,000
21,500,000
5,000,000
7,600,000
4,600,000
P43,000,000

KGY Company had the following outstanding debt as of December 31, 2013 and December 31, 2014:
11% 4-year note with simple interest payable annually on
March 31 (specifically used to fund the construction)
12% 5-year loan with simple interest payable annually on
June 30
10% 3-year loan with simple interest payable annually
on July 31

P15,000,000
20,000,000
11,000,000

How much interest should be capitalized by KGY Company for the year ended December 31, 2014?
A.
B.
C.
D.

P4,854,700
P3,076,303
P5,150,000
P3,092,170

Solution: B is correct
Date
January 1
March 1
July 1
September 1
December 1

Expenditure
Fraction
4,300,000
12/12
21,500,000
10/12
5,000,000
6/12
7,600,000
4/12
4,600,000
1/12
Average expenditures

Average expenditures
Applicable to specific borrowing
Applicable to general borrowings

Average
P4,300,000
17,916,667
2,500,000
2,533,333
383,333
P27,633,333
P27,633,333
(15,000,000)
P12,633,333

12% 5-year loan with simple interest payable annually on June 30


10% 3-year loan with simple interest payable annually on July 31
General borrowings
Average capitalization rate (3,500,000 31,000,000)
Interest on specific borrowing (15,000,000 11%)

Principal
P20,000,000
11,000,000
P31,000,000

Interest
P2,400,000
1,100,000
P3,500,000

11.29%
P1,650,000
Page 37 of 45

Interest on general borrowing (12,633,333 11.29%)


Total capitalizable interest

1,426,303
P3,076,303

5. JKL Company, on adoption of PAS 41, Agriculture, has reclassified certain assets as biological assets in its
2011 Statement of Financial Position. The total value of the entitys biological assets as of December 31,
2013 was P2,000,000 comprising:
Fruit-bearing trees
Land under trees
Roads near the plantation

P1,700,000
200,000
100,000

On February 28, 2014, the owner of the entity purchased P300,000 worth of farm animals from his friend
to be used in the business. However, such animals do not have a quoted market price, and is not traded in
an active market. Other valuation methods are also clearly inappropriate or unworkable. The estimated
lives of the trees and the animals are 20 years and 5 years, respectively.
During the last quarter of the year 2014, the following events occurred:
Increase in fair value in 2012 due to growth of trees
Decrease in fair value due to harvest
Harvest of fruits
Estimated fair value of the animals based on the judgment of the entitys CPA

P100,000
90,000
500,000
500,000

As of the December 31, 2014 Statement of Financial Position, how much should be presented as
Biological Assets by JKL Company?
A
.
B.
C.
D
.

P1,960,000
P2,010,000
P2,210,000
P1,950,000

Solution: A is correct
Fair value of fruit bearing trees, December 31, 2013
Decrease in fair value due to harvest
Increase in fair value to due to growth of trees
Fair value of fruit bearing trees, December 31, 2014
Farm animals, February 28, 2014
Depreciation of farm animals
(P300,000/5) 10/12
Farm animals, December 31, 2014
Fruit bearing trees
Farm animals
Total

P1,700,000
(90,000)
100,000
P1,710,000
P300,000
(50,000)
P250,000
P1,710,000
250,000
P1,960,000

6. Lex Company shipped inventory on consignment to Lionel Company that cost P20,000. Lionel Company
paid P500 for advertising that was reimbursable from Lex Company. At the end of the year, 70% of the
inventory was sold for P30,000. The agreement states that a commission of 20% will be provided to
Lionel Company for all sales. What amount of inventory on consignment remains in the balance sheet of
Lex Company?
Page 38 of 45

A. 0

B. 4,800

C. 6,000

D. 6,150

Solution: C is correct
Cost of inventory sent out on consignment
20,000
Multiply by: % of goods in the hand of consignee
30%
Inventory on consignment
6,000 C
7. On January 1, 2014, AJ Corporation purchased several pieces of inventory for P20,000. However, SC
Company, the seller agreed to wait for exactly two years before receiving payment. Then, on December
31, 2014, AJ Company sells the entire said inventory to BY Corporation for P30,000. AJ Company agrees
to wait for exactly three years to receive the P30,000 payment. A reasonable interest rate for these
transactions is 8% although no separate cash interest is to be paid on either the purchase or the sale. The
gross profit that AJ Company should recognize for the year ended December 31, 2014 is (Round off
present value factors to 5 decimal places):
A. 10,000

B.

7,938

C.

6,668

D.

3,815

Solution: C is correct
Sales
Less: Cost of sales
Gross Profit

30,000 x PVF P1 @ 8% for 3 periods


20,000 x PVF P1 @ 8% for 2 periods

0.79383
0.85734

23,815
17,147
6,668 C

8. In December 2014, Judith Company began including one coupon in each package of candy that it sells and
offers a toy in exchange for P5 and 5 coupons. The toys cost P12 each and an additional P4 to deliver it to
customers. Eventually, 80% of the coupons will be redeemed. During December, Judith Company sold
220,000 packages of candy, 70,000 coupons were already sent for redemption of which 20,000 is still
under processing by year end.
The estimated liability in the December 31, 2014 balance sheet of Judith Company is
A. 233,200

B.

277,200

C.

330,000

D. 387,200

Solution: B is correct
Total coupons issued
Multiply by:
Coupons expected to be redeemed
Less: coupons received and processed (70,000 - 20,000)
Remaining coupons expected to be redeemed (and processed) in the future
Divide by: conversion rate toys/coupons
Expected number of toys to be given away
Multiply by: cost per unit (12 + 4 -5)
Total estimated liability 12/31/10

220,000
80%
176,000
50,000
126,000
5
25,200
11
277,200 B

9. Barry Company purchased equipment by making a down payment of P4,000 and issuing a note payable for
P18,000. A payment of P6,000 is to be made at the end of each year for three years. The applicable rate of
interest is 8%. Shipping charges for the equipment were P2,000, and installation charges were P3,500.
How much is the capitalized cost of the equipment (Round off present value factors to 2 decimal places)?
A. 19,480

B. 21,480

C. 24,980

D. 27,500
Page 39 of 45

Solution: C is correct
Down payment
Add: PV of future installments @ 8%
Shipping cost
Installation cost
Capitalized cost of equipment

(6,000 x 2.58)

4,000
15,480
2,000
3,500
24,980

10. D Company had the following deferred tax balances at reporting date - Deferred tax assets, P1,200,000;
Deferred tax liabilities, P3,000,000. Effective from the first day of the next financial period, the company
rate of income tax was reduced from 40% to 30%. The adjustment to income tax expense to recognize the
impact of the tax rate change is:
A. Dr 600,000

B. Cr 600,000

C. Dr 450,000

D. Cr 450,000

Solution: D is correct
Tax rate
DTA
40%
1,200,000
30%
900,000
Net change (decrease)
Practical Accounting 2

DTL
3,000,000
2,250,000

Net DTL amount


= 1,800,000
= 1,350,000
(450,000)(D)

1. George Company works on a Php70 million contract in 2014 to construct an office building. During 2014,
George Company uses the cost to cost method. As of December 31, 2014, the balances in certain accounts
were: Construction in Progress, Php24.5 million; Accounts Receivable, Php2.4 million; and Progress Billings,
Php12 million. The estimated future costs to complete the project totaled Php31.85 million as of December
31, 2014.
The actual cost incurred in 2014 was
A Php17.00 million
.
B. Php24.50 million
C. Php17.15 million
D Php70.00 million
.
Solution: C is correct
Construction in Progress
Total Contract Price
Percentage of completion
Estimated future costs to complete
Percentage yet to be completed
Total estimated cost
Estimated future costs to complete
Actual cost incurred in 2014

Php24.5 million
70.0 million
35%
Php31.85 million
65%
49.00 million
31.85 million
Php17.15 million

2. Presented below are items taken from the unadjusted trial balances of Progressive Company and its branch on
December 31, 2014.
Home Office Books Branch Books
Page 40 of 45

Shipments to branch
Allowance for overvaluation of branch inventory
Shipments from home office
Purchases (from outsiders)
Merchandise inventory, January 1
Merchandise inventory, December 31
Sales
Expenses

Php300,000
99,900
Php390,000
144,600
54,600
48,750
540,000
51,000

It is the companys policy to bill all branches for merchandise shipments at 30% above cost. How much of the
branch inventory on January 1 represents purchases from outsiders?
A. Php11,700
B. Php42,000
C. Php42,900
D. Php11,000
Solution: A is correct
Allowance for overvaluation of branch inventory, December 31
Over allowance related to shipments during the year
Allowance for overvaluation of branch inventory, January 1
Merchandise inventory, January 1
Shipments from home office (9,900/30%*130%)
Merchandise inventory purchased from outsiders

Php9,900
90,000
Php9,900
Php54,600
42,900
Php11,700

3. On January 2, 2014, Polo Corporation purchased 80 percent of Seed Companys common stock for
Php216,000. Polo Corporation has assessed that it obtained control over Seed Company in accordance with
PFRS 10, Consolidated Financial Statements. Php30,000 of the excess of the consideration paid over the
book value of the net assets of Seed Company is attributed to a depreciable asset with an economic life of ten
years and the remaining balance to goodwill. On the date of acquisition, Seed reported common stock
outstanding of Php80,000 and retained earnings of Php140,000, and Polo reported common stock outstanding
of Php350,000 and retained earnings of Php520,000.
For the year ended December 31, 2014, Seed reported net income of Php35,000 and paid dividends of
Php15,000. Polo reported earnings from its separate operations of Php95,000 and paid dividends of
Php46,000. Goodwill had been impaired by Php2,000 on December 31, 2014.
How much is the consolidated net income attributable to owners of Polo on December 31, 2014?
A.
Php118,500
B.
Php118,000
C.
Php126,150
D.
Php126,000
Solution: B is correct
Purchase price
Proportionate share of NCI in book value of net assets of Seed (20% x 220,000)
Less book value of net assets of Seed Company
Excess of purchase price and NCI over the book value of net assets of Seed
Less fair value adjustment on the acquired depreciable asset
Difference attributable to goodwill

Php216,000
44,000
220,000
40,000
30,000
Php10,000
Page 41 of 45

Net income of Polo


Share in net income of Seed (80% x 35,000)
Amortization of excess of fair value over book value of the depreciable asset (30,000/10)
Impairment of goodwill
Consolidated net income attributable to owners of Polo

Php95,000
28,000
(3,000)
(2,000)
Php118,000

4. The Tommy Store began operations by selling wholesale merchandise on an installment basis and uses the
installment method of accounting. Terms include down payment of 20% and balance payable in three years;
50% in the year of sale; 30% in the year after; and 20% in the third year. Tommy includes a 25% mark-up on
cost for its selling price. Installment sales reported by Tommy are Php550,000 in 2010, Php770,000 in 2013,
and Php908,000 in 2014.
How much is the installment accounts receivable and the unrealized gross profit at the end of 2014?
A Php486,400; Php97,280
.
B. Php574,400; Php121,600
C. Php97,280; Php486,400
D Php121,600; Php574,400
.
Solution: A is correct
2013 Php770,000*80%*20%
2014 Php908,000*80%*50%
Installment receivable, end of 2012
Unrealized gross profit (Php486,400/125%*25%)

Php123,200
363,200
Php486,400
Php97,280

5. Yam Company has used a traditional cost accounting system to apply quality control costs uniformly to all
products at a rate of 14.5% of direct labor cost. Monthly direct labor cost for Atswete make-up is Php27,500.
In an attempt to distribute quality control costs more equitably, the company is considering activity-based
costing. The monthly data shown in the chart below have been gathered for Atswete make-up:
Activity

Cost Driver

Incoming material inspection Type of material


In-process inspection
Number of units
Product certification
Per order

Cost Rates
Php11.50 per type
Php0.14 per unit
Php77 per order

Quantity for
Atswete
12 types
17,500 units
25 orders

The monthly quality control cost assigned to Atswete make-up using activity-based costing is how much
higher or lower than the cost using the traditional system?
A Php8,500.50 higher than the cost using the traditional system
.
B. Php525.50 lower than the cost using the traditional system
C. Php8,500.50 lower than the cost using the traditional system
D Php525.50 higher than the cost using the traditional system
.
Solution: D is correct
Traditional Costing (Php27,500 x 14.5%)
Activity Based Costing
Incoming material inspection (Php11.50 x 12)
In-process inspection (Php0.14 x 17,500)

Php3,987.50
Php138.00
2,450.00
Page 42 of 45

Product certification (Php77 x 25)


Difference

1,925.00

Php4,513.00
(Php525.50)

6. C and D are partners sharing profits and losses in the ratio of 6:4, respectively. On January 1, the partners
decided to admit T as a new partner upon his investment of Php96,000. On this date, the interest in the
partnership of C and D are as follows: C, Php138,000; D, Php111,600. Assuming that the new partner is given
1/4 interest in the firm. The agreed capital of the partnership is Php360,000.
The admission of a new partner will result to which of the following:
A. Goodwill is Php20,400.
B. Bonus from D is Php2,400.
C. Bonus to T is Php6,000.
D. Capital balance of C after admission of T to the partnership is Php150,240.
Solution: D is correct

Capital before admission of T


Investment by T
Goodwill
Bonus to old partners
Capital after admission of T

C
Php138,000

D
Php111,600

8,640
3,600
Php150,240

5,760
2,400
Php119,760

T (25%)
Php
96,000
(6,000)
Php90,000

Total
Php249,600
96,000
14,400

Php360,000

7. The Rockwell Company of Makati opened a branch at Cebu on January 1, 2014 to expand the market of its
product. Merchandise shipped during 2014 to the Cebu branch totaled Php59,000, and this included a profit
of 25% based on cost. At the end of the year, the inventory was Php6,000 at billed price. Sales on account,
Php72,500; Expenses, Php16,500, of which Php1,200 were unpaid on December 31, 2014; Cash received
from customers on account, Php40,000, after allowing cash discounts of Php1,470; Cash remitted to the home
office during the year, Php33,000.
How much is the income of the branch in 2014 insofar as the home office is concerned?
A Php13,600
.
B. Php12,130
C. Php1,530
D Php3,000
.
Solution: B is correct
Sales
Cash discounts
Cost of goods sold [(Php59,000-Php6,000)/125%]
Expenses
Branch income

Php72,500
(1,470)
(42,400)
(16,500)
Php12,130

8. HIJ Company filed a voluntary bankruptcy petition on September 30, 2014 and the statement of affairs
reflects the following amounts:
Assets
Assets pledged with fully secured creditors
Assets pledged with partially secured creditors
Free assets

Carrying Amount

Fair Value

Php910,000
511,875
1,137,500

Php1,080,625
341,250
796,250
Page 43 of 45

Liabilities
Liabilities with priority
Fully secured creditors
Partially secured creditors
Unsecured creditors

113,750
739,375
568,750
1,478,750

Assume that the assets are converted into cash at the estimated fair values and the business is liquidated. How
much should partially secured creditors receive?
A Php341,250
.
B. Php511,875
C. Php477,750
D Php568,750
.
Solution: C is correct
Assets pledged with partially secured creditors
Share in free assets [60% x (568,750 - 342,250)]
Total cash to be received by partially secured creditors

Php341,250
136,500
Php477,750

Excess assets pledged with fully secured creditors (1,080,625 - 739,375)


Free assets
Total free assets
Less liabilities with priority
Net free assets
Total unsecured liabilities (568,750 - 342,250 + 1,478,750)
Expected recovery rate

Php341,250
796,250
1,137,500
(113,750)
1,023,750
1,706,250
60.0%

9. ABC Company, engaged in furniture business around the globe, received an order from a British Corporation
under the term of FOB shipping point on December 1, 2014. This was billed for 98,000 UK pounds and
payable on January 31, 2015. The furniture was shipped and invoiced on December 12, 2014. This shipment
was received by the customer on December 25, 2014.
Relevant spot exchange rates for UK pound to Php on various dates follow:
December 1, 2014
December 12, 2014
December 25, 2014
December 31, 2014
January 31, 2015

Buying Spot Rate


Php68.79
68.82
68.75
69.01
68.95

Selling Spot Rate


Php68.85
68.91
68.80
69.11
69.02

How much is the foreign exchange gain or loss to be reported on this transaction in 2014 income statement of
ABC Company assuming that the functional currency of ABC Company is Philippine Peso?
A Php18,620 gain
.
B. Php8,820 loss
C. Php19,600 loss
D Php19,600 gain
.
Solution: A is correct
Page 44 of 45

Buying spot rate as of December 31, 2012


Less: Buying spot rate as of December 12, 2012
Difference
Multiply: Amount billed
Foreign exchange gain

Php69.01
68.82
0.19
98,000
Php18,620

10. Government Agency AAA issued purchase order for the acquisition of office equipment costing Php30,000.
The equipment was received with the charge invoice and was paid by check after withholding taxes of 10%.
Government Agency AAA remitted the tax withheld to BIR thru a government depository bank.
What is the entry of Government Agency AAA to record payment?
A
.
B.
C.

D
.

Accounts payable
Cash National Treasury MDS
Office equipment
Cash National Treasury MDS
Office equipment
Due to BIR
Cash National Treasury MDS
Accounts payable
Due to BIR
Cash National Treasury MDS

Answer:
D Accounts payable
.
Due to BIR
Cash National Treasury MDS

27,000
27,000
30,000
30,000
30,000
3,000
27,000
30,000
3,000
27,000
30,000
3,000
27,000

Page 45 of 45

You might also like